You are on page 1of 10

NUSAMO Mock Olympiad Solutions

August 7th and 14th, 2015

1. In acute triangle ABC, M is the midpoint of AB, O is the circumcenter, and H is


the orthocenter. Let Oc be the reflection of O over line AB, and the circle centered at
Oc through B and the circle with diameter HM intersect at two points. Prove that one
of these points lies on the C-median.
C

Oc

H0
Solution 1. Note that AOc B = AOB = 2C and AHB = 180 (90 B + 90
A) = A+B = 180 C, so H lies on the circle centered at Oc through B and A. Let
H 0 be the point such that ACBH 0 is a parallelogram. Since AH 0 B = ACB = 180
AHB, AHBH 0 is cyclic. Then HBH 0 = HBA + H 0 BA = 90 A + A = 90
thus H 0 is the antipode of H on (AHB). Let T be the intersection of the C-median with
(AHB). Then HT H 0 = HT M = 90 which implies that T lies on the circle with
diameter HM .
Solution 2. Note as before that H lies on the circle centered at Oc through B and A.
Let CH AB = F and let T be the second intersection of the C-median with the


circumcircle of HF M ; invert about C with radius CH CF . Remark that that the
circle AHB inverts to the nine-point circle of ABC, and the C-median and circumcircle
of HF M invert to themselves. These three objects pass through M , so inverting back
yields the desired conclusion.
Solution 3 (by blasterboy). Note as in Solution 1 that H lies on the circle centered at
Oc through B and A. Then if the other intersection of the two circles, apart from H, is
T , then we wish to show C, T, M are collinear. Let F = CH AB. Then it suffices to
show F T M + F T C = 180.
Noting that F HT M is cyclic, F T M = F HM . Furthermore, HM T = T F H =
4CHM 4CT F = F T C = CHM
Now note that F T M + F T C = F HM + CHM = 180 since CF is an altitude
passing through orthocenter H.
Solution 4 (by djmathman). Remark that the circumradius of the circle centered at Oc
and passing through B is R. In addition, recall that the reflection of H over AB lies on
the circumcircle of 4ABC, so the circumradius of 4AHB is also R. Now let T be the
foot of the perpendicular from H to CM ; it suffices to show that AHT B is cyclic.
Let D = AH BC. Remark that since CDH = CT H = 90 , quadrilateral
CDT H is cyclic. This means that, among other things, CT D = CHD = CBM ,
so 4CDT 4CM B = BM 2 = M T M C.
Now let T 0 be the reflection of T over M . Then the recently-discovered metric relation
becomes AM BM = M C M T 0 . By reverse Power of a Point, T 0 (ABC). Finally,
since AM = M B and T M = M T 0 , AT BT 0 is a parallelogram, so 4AT 0 B = 4BT A.
Hence the circumradius of 4AT B is also R, implying the desired cyclicity.

2. Prove that for all n N,


2 1
nX

k=1

4n3 n 3
6

Solution 1. Note that if r, a N, then


p
a
r2 + a r +
2r

a 2
(squaring both sides and subtracting gives 2r
0, which is true by the Trivial
inequality). Then, note that
2 1
nX

k=

2 1
nX

(b kc + { k})

k=1

k=1

For any integer k, with 1 k n 1, there are obviously 2k + 1 integers m for which

b mc = k (since (k + 1)2 k 2 = 2k + 1). Thus,


2 1
nX

b kc =

n1
X

k(2k + 1) =

k=1

k=1

4n3 3n2 n
.
6

Now consider the fractional part. We concern ourselves with summing this from k 2 to
(k + 1)2 1, then summing across all values of k. Since
p
a
k k2 + a k +
2k

a
we know that { k 2 + a} 2k
. Then
(k+1)2 1

X
m=k2

2k
X

a
1
{ m}
=k+ .
2k
2
a=0

Summing over all values of k gives


2 1
nX

{ w}

w=1

n1
X
k=1

1
k+
2


=

n2 1
2

which is an upper bound on the sum of the fractional parts. Adding this to the sum of
the floors gives the desired bound.
Solution 2 (by blasterboy). We induct on n. If n = 1, then we have 0 = 0. Now assume
for some m we have
2 1
m
X

k=1

k=

1 + ... +

p
4m3 m 3
m2 1
6

It suffices to show that


(m+1)2 1

k=1

2 +2m
mX

1 + ... +

k=1

But note that

p
4(m + 1)3 (m + 1) 3
m2 + 2m
6

p
1 + ... + m2 + 2m
p
p
p

4m3 m 3 2
= [ 1+...+ m2 1]+[ m2 +...+ m2 + 2m]
+[ m +...+ m2 + 2m]
6
so it suffices to show that
p

(2m + 1)2
4(m + 1)3 (m + 1) 3 4m3 m 3

=
m2 + ... + m2 + 2m
6
6
2
Note that by Cauchy-Schwarz, we have
p

p
(1 + ..... + 1)(m2 + .... + m2 + 2m) m2 + ... + m2 + 2m
p

p
= (2m + 1)2 (m2 + m) m2 + ... + m2 + 2m
Furthermore, the following holds true:
0 1 = 4m2 +4m (2m+1)2 =

p
p
2m + 1
(2m + 1)2
m2 + m
= (2m+1) m2 + m
2
2

Therefore we have

m2 + ... +

m2 + 2m

p
(2m + 1)2
(2m + 1)2 (m2 + m)
2

This concludes our argument.

3. The points on a n n lattice grid are colored one of k colors: A1 , A2 , . . . , Ak . A


rectangle whose vertices lie on the grid and whose sides are parallel to those of the grid
is called happy if for some r, all of its vertices are of color Ar .
a) Show that if n > k(k + 1), there must exist a happy rectangle.
b) Show that if n > k(k + 1), there must exist at least


k+1 2
2

happy rectangles.

Solution. Let Ar be the most common color in the grid. By the pigeonhole principle,
2
there are at least Nr = d nk e vertices of color Ar . Denote by f (i, r) the number of points
of color Ar in the ith column of the grid. Then we know that
Nr =

n
X

f (i, r)

i=1

Consider a graph Gr , whose vertices are the points of color Ar , and whose vertices are
connected only if they lie in the same column. Then

n 
X
f (i, r)
E(Gr ) =
2
i=1

where E(Gr ) denotes the number of edges in Gr . Note that

! n
!2
n
n
X
X
X
f (i, r)2
12
f (i, r)
i=1

by Cauchy-Schwarz. Then

j=1

n
X

i=1

f (i, r)2

i=1

Nr2
.
n

We established that
E(Gr ) =


n 
X
f (i, r)
i=1

1X
1
(f (i, r)2 f (i, r))
2
2
i=1


Nr2
Nr .
n

Call two edges in Gr equivalent if they lie in the same relative positions within their
columns. Note that there are n2 equivalence classes in Gr . We now follow the same
logic as before. Label these equivalences classes R1 , R2 , R3 , . . . , R(n) . Then we know
2
that
n
(2)
X
E(Gr ) =
|Ri |
i=1

Note that for any two elements in the same equivalence class, the four vertices of the
two edges form a happy rectangle. Then there are
(n2 ) 

X
|Ri |
2

i=1

happy rectangles. Applying the similar manipulations as before using the CauchySchwarz inequality, we see that there are at least
!
1 E(Gr )2
 E(Gr )
n
2
2
happy rectangles. We know that
1
E(Gr )
2

Nr2
Nr
n

 
1 n2
=
2 k

d nk e
1
n

n2
k



 n2 (n k)
n
1 =
k
2k 2

Let H denote the number of happy rectangles.


!
E(Gr )2
 E(Gr )
n

1
H
2

=
=

n2 (nk)

2


2k2

n
2


k)

2k 2

n2 (n



n2 (n k) n2 (n k)
1
4k 2
k 2 n(n 1)


n2 (n k) n(n k)
1
4k 2
k 2 (n 1)


n2 (n k) n k
1
4k 2
k2
n2 (n k)(n (k 2 + k))
4k 4

Note that if n > k 2 + k, there is then at least one happy rectangle. Furthermore, letting
n k 2 + k + 1, we see that
H

(k 2 + k + 1)2 (k 2 + 1)(1)
(k 2 + k)2 (k 2 )
(k + 1)2

=
4k 4
4k 4
4

as desired.

4. Call an n-sided nondegenerate convex polygon multi-Pythagorean if its n angles


a1 , a2 , . . . , an1 , an satisfy
a21 + a22 + + a2n1 = a2n
(here all ai are in degrees and n 6= 4). Find all multi-Pythagorean polygons whose angles
are all integers.
Solution. By Titus Lemma, we have that
a21 + a22 + + a2n1

(a1 + a2 + + an1 )2
n1

= a2n (n 1) (180(n 2) an )2
Since both sides are positive for n > 2, we can take the square root to obtain

an n 1 180(n 2) an

= an 180( n 1 1)
For n 5, an 180 which is impossible, so it suffices to check the case n = 3.
For n = 3, we have that
a21 + a22 = (180 (a1 + a2 ))2
which reduces to
(180 a1 )(180 a2 ) = 16200
We want 0 a1 , a2 180 which is equivalent to 0 180 a1 , 180 a2 180, so we
seek numbers m and n such that mn = 16200 and m, n 180. Looking through the
factors of 16200 = 180(90), we see that the only such pairs are (100, 162), (135, 120),
(150, 108) (and permutations). Then if WLOG a1 a2 , we see that the only ordered
triples (a1 , a2 , a3 ) which are multi-Pythagorean are (18, 80, 82), (30, 72, 78), (45, 60, 75).
In summary, the only multi-Pythagorean polygons are those with angles (18, 80, 82),
(30, 72, 78), (45, 60, 75) and appropriate permutations.

5. Let ABC be a triangle, and define X to be the intersection of the external angle
bisectors of B and C and I to be the intersection of the internal angle bisectors of
B and C. Let M be the circumcenter of 4BIC, and let G be the point on BC such
that XG BC. Construct a circle with diameter AX. If and the circumcircle of
4ABC intersect at a second point P , and the altitude from A onto BC intersects at
H, prove that H, M , G, and P are collinear.
A

D G

Solution 1. Let R be the tangency point of the A-excircle on AB and S the tangency
point on AC. By Miquels Theorem, there is a spiral similarity that carries 4P BR to
BR
BG
4P CS, so PP B
C = CS = CG . By the Angle Bisector Theorem, this implies that G lies
on P M , solving half the problem.
To prove that H lies on this line, it suffices to prove that AP M = AP H. By
angle chasing, we get AP M = ACM = C + BCM = C + BAM = C + A
2 .
Similarly, AP H = AXH. Let AH BC = K; it is clear that 4AKD 4AHX,
A
where D is AX BC; we get that AXH = ADK = 180 B A
2 = C + 2 ,
and were done.
Solution 2. To prove the collinearity of M, G and P , invert about M with radius M B.
It is well-known that M is the midpoint of minor arc BC in the circumcircle of 4ABC
and that A, I, M and X are collinear. Note that and the circumcircle of 4BIC pass
through X by Fact 5 and their centers are collinear through this point, so they are

tangent. This inversion sends line M G to itself, the circumcircle of 4ABC to line BC,
and to the circle with diameter DX. Remark that DGX = 90 , so G lies on the
inverted circle and all three objects pass through this point, implying the conclusion.
Now proceed as in Solution 1.

6. A sequence of positive integers a1 , a2 , a3 , . . . , an has the property that every term is


a distinct power of 2. Let

( j
!
)

Y

S=
ak 1 i j .

k=i

Suppose S does not contain a multiple of n + 2. Find all possible values of n.


Solution. Consider when n is even. Then given a sequence hai i not containing 1, all
elements of S are odd so it is impossible that S contains a multiple of n + 2 which is
even. Now we consider when n is odd. Let f (k) = ordk+2 (2); evidently, there are f (n)
possible values of ai modulo n + 2. Suppose n f (n). We claim that in this case, every
sequence ai gives a set S containing a multiple of 2. Denote
Pi,j =

j
Y

ak .

k=i

There are at most n different possible values of P1,j . If they are all pairwise not congruent
modulo n + 2, then since n f (n), by the Pigeonhole Principle one of them is congruent
to 1 (mod n + 2) and were done. Otherwise assume for some j1 6= j2 , it is true that
P1,j1 P1,j2 (mod n + 2). Then
j1
Y

ak

k=1

j2
Y

ak

(mod n + 2)

k=1

WLOG let j1 > j2 . n + 2 is odd and the LHS and RHS are both powers of 2, so we can
safely say
j1
Y
ak 1 (mod n + 2)
k=j2

so then Pj2 ,j1 1 is a multiple of n + 2 and we are done. Thus, if n f (n), we


have shown that regardless of the sequence hai i, S contains a multiple of n + 2. Now
we consider n < f (n). Since f (n) = ordn+2 (2), by Eulers Theorem we know that
f (n) (n + 2) n + 1 since the maximal value would occur at primes. If n < f (n),
then n < f (n) n + 1 = f (n) = n + 1. Thus n + 2 is prime and 2 is a primitive
root. Let p = n + 2 then be a prime. We must show that for these values of p there
exists a sequence hai i for which S contains no multiples of n + 2. Consider the sequence
ak = 2k(p1)+1 = 2kn+k+1 . These are all equivalent to 2 (mod n + 2), and since 2 is a
primitive root modulo n+2, it is impossible that multiplying any consecutive subsequence
of these will be congruent to 1 (mod n + 2). Then our final solution set is
n {2k | k N} {p 2 | p is a prime for which 2 is a primitive root}.

10

You might also like